Hello fellow Brilliantians!
As most of you know, the IMO is one of the biggest and most famous of all international Maths Olympiads. Following are the problems which appeared in the test at the end of the First Training Camp of Pakistan for the IMO 2017. Enjoy!:
Let be a triangle. Let be the feet of the perpendiculars from to , to and to respectively. Let and be the feet of the perpendiculars from to and respectively. Prove that are collinear.
Find the number of ordered pairs of sets such that .Compute the answer for .
Note:
and are the same pair if and .
or can be an empty set (the pairs in which either one of is an empty set are to be counted as well)
Let be two positive integers (not necessarily distinct) and let be a prime such that .Prove that
Note:
denotes the Least Common Multiple of the numbers and .
Find all polynomials with real coefficients such that the polynomial is constant.
Easy Math Editor
This discussion board is a place to discuss our Daily Challenges and the math and science related to those challenges. Explanations are more than just a solution — they should explain the steps and thinking strategies that you used to obtain the solution. Comments should further the discussion of math and science.
When posting on Brilliant:
*italics*
or_italics_
**bold**
or__bold__
paragraph 1
paragraph 2
[example link](https://brilliant.org)
> This is a quote
\(
...\)
or\[
...\]
to ensure proper formatting.2 \times 3
2^{34}
a_{i-1}
\frac{2}{3}
\sqrt{2}
\sum_{i=1}^3
\sin \theta
\boxed{123}
Comments
Problem 4
I think my solution is correct. Please check if it is or not.
By observation, if P(x)=c, then Q(x) is also a constant function. If P isn't constant, we can do as follows:
We can assume P is monic since if it wasn't, we can divide out by the leading coefficient to still have a constant on the RHS. Let P(x)=(x−r1)(x−r2)…(x−rn). By Vieta's Formula,
P(x)⟹(x−1)P(x)P(x−1)⟹(x+1)P(x−1)=xn−(r1+r2+…+rn)xn−1+…=xn+1−(r1+r2+…+rn+1)xn+…=(x−r1−1)(x−r2−1)…(x−rn−1)=xn−(r1+r2+…+n)+…=xn+1−(r1+r2+…+rn+n−1)xn+…
The coefficients for the xn in the expression for (x+1)P(x) and (x−1)P(x) must cancel each other out, so they must be equal. Thus, r1+r2+…+rn+1=r1+r2+…+rn+n−1, so n=2. Thus, P(x)=x2−(r1+r2)x+(r1r2). We now have (from above)
(x−1)P(x)P(x−1)⟹(x+1)P(x−1)=x3−(r1+r2+1)x2+(r1+r2+r1r2)x−r1r2=x2−(r1+r2+2)x+(r1r2+r1+r2+1)=x3−(r1+r2+1)x2+(r1r2−1)x+(r1r2+r1+r2+1)
The coefficient for x in the expression for (x+1)P(x) and (x−1)P(x) must cancel each other out, so they must be equal. Thus, r1+r2+r1r2=r1r2−1, so r1+r2=−1. Substituting back into P(x), we get P(x)=x2+x+k.
Checking, we have P(x−1)=x2−x+k, so (x+1)P(x−1)=x3+(k−1)x+k whereas (x−1)P(x)=x3−(k−1)x−k. Subtracting these two, we get Q(x)=2k, which is a constant. Therefore, our solution has been verified.
Therefore, all solutions that satisfy are P(x)=c and P(x)=A(x2+x+k), where c, A and k are constants.
Log in to reply
My solution is exactly the same in the sense that it computes the coefficent of xn in Q(x) but I used the binomial expansion after using a generic polynomial
P(x)=∑i=0naixi
Expanding and comparing coefficents kills it
Problem 3
I found this problem pretty trivial.
First, we will prove the gcd(a,a+p) is either 1 or p. Let k be the gcd. We have
kk⟹k⟹k∣a∣a+p∣a+p−a∣p
Thus, the gcd is either 1 or p. The same goes for b and b+p. We now have three cases:
Case 1: gcd(a,a+p)=gcd(b,b+p)=1
Thus, their LCM for each is equal to their product, so a(a+p)=b(b+p), which simplifies as a2−b2=p(b−a) so −(a+b)=p or b−a=0. The former cannot be true since a,b,p are all positive integers. Therefore, b−a=0 so a=b.
Case 2: gcd(a,a+p)=gcd(b,b+p)=p
Thus, their LCM for each is equal to their product divided by p, so pa(a+p)=pb(b+p), which simplifies to the statement in the previous case. Thus, a=b.
Case 3: gcd(a,a+p)=1, gcd(b,b+p)=p
Thus, each of their lcm's are a(a+p) and pb(b+p). Therefore,
ap(a+p)=b(b+p) However, we know that p∣b and p∣b+p. Thus, p2∣b(b+p), which means p2∣ap(a+p), so p∣a(a+p). However, this means either p∣a or p∣a+p, which is contradictory since either statement results in the other being also true, so their gcd=p=1. Thus, we have a contradiction, so this case cannot occur.
Therefore, over all the cases a=b.
Log in to reply
This is pretty trivial...
Log in to reply
Exactly! But I wanted to case bash one of the problems.
Problem 2
This was a fairly simple problem.
Let S be the set {1,2,…n}. Let A={a1,…,ak} with k≤n. Then, B must contain at least {S/A} as well as any element chosen from A. This gives us 2k choices since each element in A may or may not be in B. The number of ways to choose k elements in A for this to satisfy is (kn). Thus, we must sum (kn) ×2k from k=0 to n.
k=0∑n(kn)2k=k=0∑n(kn)2k1n−k=(2+1)n=3n
Thus, the general formula is 3n, so if n=5, 3n=243.
Log in to reply
Here is another solution I found:
Note that we have exactly 3 choices for each element, It goes in A,B or A and B. Since the choices for each element is exclusive we have that the number of ways is exactly 3n
Log in to reply
Yep, that works as well, and is fairly quick. I saw my way first but this is an even quicker solution.
Problem 1 is from India's RMO 2006. Check the solution here
Log in to reply
The question isn't too hard. Angle chasing is all it takes.
Log in to reply
I found at least 2 solutions for this:
A nicer approach, Consider the triangle BFH. PQR is its simson line. Similarly QRS is collinear and we are done!
Log in to reply
Problem 1
This was fairly trivial, with a good diagram.
Firstly, since DP is perpendicular to AB and CF was perpendicular to AB, DP and CF are parallel. Similarly, DS and BE are parallel. Also, since APDS and BFEC are cyclic, we have ∠APS=∠ADS=∠ECB=∠EFA, Thus, PS and EF are parallel.
Note that QPBD is cyclic, as well as EFBC. Since they both share angle ABC, both P and F are on AB, and D and C are on BC, it follows that EF is parallel to PQ. From the previous result, we get that P,Q,S are collinear and parallel to EF. Using a similar argument for R, we get P,Q,R,S are all collinear parallel to EF.
Alternate solution (Kudos to Sualeh Asif)
This is the slick solution.
Since HFBD is cyclic, where H is the orthocentre, and DP,DQ,DR are perpendiculars to BF, BH and HF respectively, PQR is the Simson Line of HBF (D is the point on the circumcircle). Thus, P,Q,R are collinear. Similarly, Q,R,S are collinear so we are done.
sigma upper limit 2001 and lower limit k=1 (x+k)^2=y^2 has no integer solutions prove this was a problem at the imo training camp at Romania Please help me with the solution